PT45.S4.Q18 - decentralization enables divisions

CrushLSATCrushLSAT Member
edited October 2016 in Logical Reasoning 150 karma
https://7sage.com/lsat_explanations/lsat-45-section-4-question-18/
I'm pulling my hair out as I am trying to figure out why the correct answer for PT 45 S4 Q18 is A... The question reads,

18. Decentralization enables divisions of a large institution to function autonomously. This always permits more realistic planning and strongly encourages innovation, since the people responsible for decision making are directly involved in implementing the policies they design. Decentralization also permits the central administration to focus on institution-wide issues without being overwhelmed by the details of daily operations.

Answer:
A) In large institutions whose divisions do not function autonomously, planning is not maximally realistic.

Ok, first of all, is the first sentence referring to a conditional relationship or a causal relationship? I assumed it was the former and
created a conditional chain that looks like this: D --> A. If this is the case, the next sentence can be connected with the previous condition by saying divisions working autonomously always permits more realistic planning like this: D -> A -> MRP (the word "always" is a necessary condition indicator word and I assumed this is how I should interpret this sentence).

To me answer choice A sounds like a mistaken negation where we are saying ~A -> ~MRP... and I was really confused by this. How can we assume that a large institution's planning is not maximally realistic just be cause their divisions do not function autonomously? Or am I interpreting this AC incorrectly and it is really saying ~MRP -> ~A ...

I am always confused as to how to correctly identify a conditional or causal relationship... For example, when they start talking about how decentralization "permits" the central administration to focus on ... how would you define this relationship as? Could anyone please shed some light on how you would go about solving this question? Many thanks in advance!

Comments

  • TheLSATTheLSAT Member
    edited October 2016 301 karma
    The key to answer choice "A" is a combination of the 1st and 2nd sentence. The stimulus states that decentralization enables a large institution to function autonomously, which in turn "ALWAYS" permits "MORE" realistic planning. You can infer from what I just said that in large institutions that are NOT decentralized (i.e. the ones that do not function autonomously), planning is not realistically maximal because once the institutions become decentralized, there is ALWAYS room to make planning "MORE" realistic (2nd sentence of the stimulus). I hope this helps.
  • CrushLSATCrushLSAT Member
    150 karma
    @"A. Mathews" Thanks for your explanation but I'm still not clear with this logic... The part where I am struggling to understand is where you said that "institutions that are NOT decentralized, planning is not maximal" because to me it sounds like we are just negating a sufficient condition (not decentralization) to come to the conclusion (not maximal). Isn't this a flaw that treats a sufficient condition as a necessary condition?
  • TheLSATTheLSAT Member
    edited October 2016 301 karma
    This answer choice can be derived at without using conditional reasoning (just intuition). Forget about conditionality and focus on the hidden assumption. If decentralization always (100% of the time) permits "more" realistic planning, there is a hidden assumption that the large institutions that are not decentralized could increase the realism of their planning. Example: Lets say there is a large institution that has not been decentralized yet that has its planning at 70%. According to the stimulus, once you decentralize it, its planning can be improved (maybe to 80%?). From all of this, I can infer that the large institution before it was decentralized (i.e. before its divisions started to function autonomously), was not at its maximal planning potential.
  • CrushLSATCrushLSAT Member
    edited October 2016 150 karma
    @"A. Mathews" Wow, I just had an ah-ha moment lol. thank you so much for that example! It's always hard for me to decide exactly when to use the conditional statements and when not to but I think I get what you're saying now. I will have to mull over this a bit more but again, thanks so much for your help :)
  • TheLSATTheLSAT Member
    301 karma
    @CrushLSAT Absolutely, I am glad I was able to help.
Sign In or Register to comment.